You are on page 1of 49

FREE NLE REVIEW QUESTIONS: NP 3 & NP 4 Med-Surg

( Taken from sample questions posted on april & late march)

The appendix is a small pouch attached to the large intestine.


Appendicitis is characterized by inflammation of the appendix. A patient with appendicitis is at risk
for developing:
A. hemorrhage
B. pulmonary edema
C. peritonitis
D. bowel obstruction
.
.
.
ANSWER: C
Peritonitis is caused by the rupture of the appendix either due to increased intraluminal pressure or
transmural swelling and inflammation. Signs of peritonitis include hyperthermia and board-like
abdominal rigidity.
Choices A and D are incorrect because they are not complications of appendicitis. Renal failure
could result if the patient develops septic shock, but it is not a direct complication of appendicitis

What common side effect should the nurse monitor in a client who just received epidural
anesthesia?
A. Fever
B. Bradycardia
C. Hypotension
D. Pallor
Answer is C
Ratio : hypotension is a common side effect of epidural anesthesia. The nurse should monitor the patient's BP in the
post-operative period.

In an elderly client who is being treated with narcotics, what should be the nurse's highest priority?
A. Assessing the client's ability to swallow
B. Checking the client's liver function
C. Monitoring the clients's serum creatinine levels
D. Assessing the client's range of motion
.
.
.
.
.
ANSWER: B
Elderly clients have slowed liver function that results in delayed excretion of the drug. This may
potentially lead to hepatotoxicity.
A client Extracellular fluid fall behind 20% of his normal body weight. The nurse finding on
assessment may include EXCEPT
A.Weight loss
B.Edema
C.Increase Output
D.Decrease Intake
#Clue #fVD #except
Correct Answer B.
Urolithiasis is the presence of calculi (stones) in the urinary tract. There are many types of renal
calculi, depending on the etiology. Your patient has calcium oxalate stones. The following should be
avoided by the patient, except:
A. apple
B. black tea
C. banana
D. spinach
.
.
.
ANSWER: C - Banana
Black tea, cocoa, citrus fruits, apples, grapes, peanuts, chocolate and spinach are foods that are
source of oxalate. Reduction of urinary oxalate content may help prevent calcium oxalate stones
from forming.

A client with refractory myasthenia gravis undergoes plasmapheresis therapy. The nurse determines
that the therapy was effective if the client demonstrates improvements in:
A. Vital capacity
B. Leg strength
C. Ptosis
D. Diplopia
.
.
.
.
.
Answer( lippincott ): A
Plasmapheresis therapy removes the anti-bodies that cause MG; therefore, the lung muscles will
function with greater strength delivering more vital capacity.
MG affects the upper limbs, and an increase in leg strength is not an outcome of plasmapheresis.
Once the MG client has symptoms of ptosis and diplopia, they will not be reversed by
plasmapheresis therapy.

From an ECG reading, a QRS complex represents


A. Atrial depolarization
B. Ventricular depolarization
C. Atrial repolarization
D. Ventricular repolarization
.
.
.
ANSWER: B
QRS complex represents ventricular depolarization. The QRS complex is normally less than
0.12seconds in duration. T wave represents the ventricular repolarization, the time when the cells
regain the negative charge; also called the resting state. P wave represents the atrial depolarization.
Atrial repolarization is not distinctly recognized because its wave has low amplitude and it occurs at
the same time as ventricular depolarization

Which assessment finding is most indicative of pericarditis?


A. Third heart sound
B. Pericardial friction rub
C. Rales auscultated at the right lower lobe
D. Pain not relieved by a change in position
.
.
.
ANSWER: B
The high-pitched, scratchy sound is heard at the left lower sternal border. It is produced by the
pericardial layers (inflamed and rough) when their surfaces rub against each other. A third heart
sound is not present in a client with pericarditis, but may be present in left-sided heart failure. The
lungs when auscultated are typically clear. Pain is worse when the client is lying down but is relieved
by sitting up and leaning forward.

A client with emphysema experiences a sudden episode of shortness of breath and is diagnosed
with a spontaneous pneumothorax. The client asks, "How could this have happened?" What likely
cause of the spontaneous pneumothorax should the nurses' response take into consideration? 
A. Pleural friction rub
B.Tracheoesophageal fistula
C.Rupture of a subpleural bleb
D.Puncture wound of the chest wall
Answer:C The etiology of a spontaneous pneumothorax is commonly the rupture of blebs on the lung surface. Blebs
are similar to blisters, but are filled with air.

On assessement the nurse observe redness on the skin and document the client has:
A.petechiae
B.rash
C.Erythema
D.Lesion
correct answer C.

your client has returned from parathyrodectomy and must be monitored for hypocalcemia. which of
the following indicators of hypercalcemia:
1. Paralysis
2.muscle weakness
3.tingling sensation
4.Muscle cramps
5. tetany
Choices:
A.1234
B12
C.135
D.124
345- are indicators of Hypocalcemia therfore 12 hypercalcemia 
Correct answer B
Peptic ulcer disease (PUD) is characterized by excess gastric acid secretion. It involves a break in
continuity of the esophageal, gastric, or duodenal mucosa. The following are characteristics of
gastric ulcer, EXCEPT:
A. Melena more common than hematemesis
B. Decreased acid secretion
C. Recurrence unlikely after surgery
D. Has a malignancy potential (occurs in approximately 10% of clients)
.
.
.
ANSWER: A
The bleeding pattern for duodenal ulcer is melena while hematemesis is for gastric ulcer. Perforation
is more likely in a duodenal and less likely in a gastric. In gastric ulcer, pain is usually caused by
food ingestion while in duodenal ulcer, food relieves pain; pain occurs 2 to 3 hours after a meal. The
best diagnostic exam is endoscopy specifically esophagastroduodenoscopy (EGD). Medications
prescribed include antacids, H2 antagonists, proton pump inhibitors (best drug to decrease the acid
secretion of the parietal cell), mucosal protective agent (sucralfate or carafate), and antibiotics (due
to infection usually caused by H. pylori).

 Which of the following parameters would the nurse monitor to evaluate the effectiveness of
thickened feedings for an infant with gastroesophageal reflux (GER)?
a. vomiting
b. Stools
c. Uterine
d. Weight
Correct Answer A. Thickened feedings are used with GER to stop the vomiting. Therefore, the nurse
would monitor the child’s vomiting to evaluate the effectiveness of using the thickened feedings. . If
feedings are ineffective, this should be noted before there is any change in the child’s weight.
The nurse is analyzing an ECG rhythm strip on an assigned client. The nurse notes that there are
three small boxes from the beginning of the P wave to R wave. The nurse records that the clientd
PR interval is
A. 0.12 second
B. 0.20 second
C. 0.24 second
D. 0.40 second
ANSWER A
standard ECG Graph paper measurements are 0.04 second for each small box on the horizontal
axis(measuring time) and 1mm (measuring voltage) for each small box on the vertical axis.
Test taking --> knowledge regarding ECG basics is necessary to answer this 

The nurse undesrtands that home healthcare is provided who are. EXCEPT
A. cant afford hospitalization
B. Chronically ill
C. Those with disabilities
D. Recuperating

Correct Ans: A

A 60 year old male client comes into the emergency department with complaints of crushing
substernal chest pain that radiates to his shoulder and left arm. The admitting diagnosis is acute
myocardial infarction (MI). Immediate admission orders include oxygen by nasal cannula at
4L/minute, blood work, a chest radiograph, a 12-lead electrocardiogram (ECG), and 2 mg of
morphine sulfate given intravenously. The nurse should first:
A. order the chest radiograph
B. obtain a 12 lead ECG
C. administer morphine
D. obtain a blood work
.
.
.
ANSWER: C
The nurse's priority at this time is to alleviate the chest pain of the client. Obtaining chest
radiograpgh, ECG and blood work are all important and can be done after administering morphine.

The American Cancer Society recommends routine screening to detect colorectal cancer. Which
screening test for colorectal cancer should the nurse recommend?
A. Carcinoembryonic antigen (CEA) test after age 50
B.Flexible sigmoidoscopy after age 30
C. Annual digital examination after age 50
D.barium enema after age 20
Correct answer C. dpat kung B sagot dpat 5 years at over age 50 si sigmoidoscopy.. Kapag A namn
sagot mo para sya sa meron ng Colorectal cancer

The skull protects the brain from injury. It is resilient and tough but a severe impact or blow can
result in fracture. Otorrhea and rhinorrhea are most commonly seen in which type of skull fracture?
A. Parietal
B. Occipital
C. Basilar
D. Temporal
.
.
.
ANSWER: C
A basilar skull fracture is a linear fracture at the base of the skull. It is related with a dural tear and it
initiates at specific points on the skull base that usually extends into the anterior, middle, or posterior
fossa and results in cerebrospinal fluid leakage from the nose or ears (otorrhea and rhinorrhea). Any
fluid suspected of being cerebrospinal fluid (CSF) should be checked for glucose. Signs associated
with basilar skull fracture are: halo sign (characterized by blood stain surrounded by yellowish stain),
raccoon's eye (periorbital ecchymosis) and battle's sign (ecchymosis over the mastoid area).

The nurse is applying ECG electrodes to a diaphoretic client. The nurse does which of the ff to keep
the electrodes from coming loose?
A. Secures the electrodes with adhesive tape
B.places clear, transparent dressings over the electrodes
C. Applies lanolin to the skin before applying the electrodes
D. Cleanses the skin with alcohol before applying the electrodes.
ANSWER D
Alcohol defats the skin and will help the electrodes adhere to skin. Placing adhesive tape or clear
dressing over the electrodes will not help the adhesive gel of the actual electrode to make better
contact with the diaphoretic skin. Lanolin or any other lotion makes the skin slippery and prevents
good initial adherence
Test taking --> use the process of elimination. Note that options A B are comparable. Note that
option D addresses cleansing the skin.
ECG is a diagnostic tool to used to evaluate the electrical activity of the heart. Which of the following
ECG results would reveal that there is myocardial ischemia?
A. ST segment elevation and peaked T wave
B. ST segment elevation and inverted T wave
C. ST segment depression and peaked T wave
D. ST segment depression and abnormal Q wave
.
.
.
ANSWER: B
The ECG reading of a client who had myocardial ischemia would reveal an inverted T wave and ST
segment elevation. The ECG tracing of a client with myocardial injury is characterized by a
symmetric and peaked T wave, and at least 1 mm ST segment elevation while myocardial infarction
will show an abnormal or pathologic Q wave that develops within 1-3 days after MI.
he nurse is admitting a client with a suspected fluid imbalance. The most sensitive indicator of body
fluid balance is:
A.daily weight.
B serum sodium levels.
C.measured intake and output.
D.blood pressure.
Correct answer A.

A client underwent cataract removal with an intraocular lens implant. The nurse is giving the client
discharge instructions. These instructions should include which of the following?
A.Avoid lifting objects weighing more than 5 lb (2.27 kg).
B. Lie on your abdomen when in bed.
C.Keep rooms brightly 
D.Avoid straining during bowel movement or bending at the waist.
Correct answer D 
if ur answer A. Dapat 15 lbs .. Kapag B dapat naka sidelying or supine C. Avoid bright can cause
IOP.

A client is to be discharged from an acute care facility following treatment for right leg
thrombophlebitis. The nurse notes that the client's leg is pain-free, without redness or edema. The
nurse's actions reflect which step of the nursing process?
A. Assessment
B. Analysis
C. Implementation
D.Evaluation
Correct answer D

A client who has recently had surgery for prostate cancer expresses to the nurse feelings of anger
toward God, his church, and the clergy. Which intervention isn't appropriate for this client?
<
A.Acknowledging the client's spiritual distress
B. Inviting the client's clergyman to visit him
C.Encouraging the client to discuss religious beliefs and practices
D. Encouraging the client to discuss concerns with the clergy
CORRECT ANSWER B. The nurse shouldn't invite his clergyman to visit the client, unless the client
specifically asks to see that member of the clergy.

A nurse is assessing a client with cardiac disease at the 30weeks gestation antenatal visit. The
nurse assess lung sounds in the lower lobes following a routine blood pressure screening. The nurse
performs this assessment to
A. Identify mitral valve prolapse
B. Identify cardiac dysrhythmias
C. Rule out the posibility of pneumonia
D. Assess for early signs of congestive heart failure
ANSWER D
Fluid volume during pregnancy peaks between 18 and 32weeks gestation. During this period it is
essential to observe and record maternal data that would indicate further signs of cardiac
decompensation or CHF in the pregnant client with cardiac disease.

The nurse is prepAring to care for a patient following ereterolithotomy who has a ureteral catheter in
place. The nurse plans to implement which action in the management of this catheter when the client
arrives from the recovery room
A. Clamp the catheter
B. Places tension on the catheter
C. Checks the drainage from the catheter
D. Irrigates the catheter using 10ml sterile normal saline
ANSWER. C
Drainage from the ureteral catheter should be checked when the client returns from the recovery
room and at least every 1to 2hours thereafter. The catheter drains urine from the renal pelvis, which
has a capacity of 3to5ml. If the volume of urine or fluid in the renal pelvis increases, tissue damage
to the pelvis will result from pressure. Therefore the ureteral tube is never clamped. Additionaly
irrigation is not performed unless there is a specific physicians order to do so.
Test taking strategy--> focus on the subject a ureteral catheter and think about the anatomy of the
kidney. Recalling that the ureteral catheter is placed in the renal pelvis and recalling the anatomy of
this anatomical location will assist in eliminAting options A B D.
REFEREnce: black&hawks (2005). Med-surgical nursing. ..

1.Which of the following definitions best describes diverticulosis?


a. An inflamed outpouching of the intestine
b. A non – inflamed outpouching of the intestine
c. The partial impairment of the forward flow of instestinal contents
d. An abnormal protrusions of an oxygen through the structure that usually holds it
2. Which of the following types of diet is implicated in the development of diverticulosis?
a. Low – fiber diet
b. High – fiber diet
c. High – protein diet
d. Low – carbohydrate diet
3. Which of the following tests should be administered to client with diverticulosis?
a. Proctosocpy
b. Barium enema
c. Barium swallow
d. Gastroscopy
...................................................................................................................
...................................................................................................................
...................................................................................................................
ANSWERS: 1)B 2)A 3)B
1. Answer: B. A non – inflamed outpouching of the intestine. 
Rationale: An increase intraluminal pressure causes the outpouching of the colon wall resulting to
diverticulosis. 
Option A suggests diverticulitis.
2. Answer: A. Low – Fiber Diet 
Rationale: A lack of adequate blood supply and nutrients from the diet such as low fiber foods may
contribute to the development of the disease.
3. Answer: B. Barium enema 
Rationale: Barium enema is used to diagnose diverticulosis, however, this is contraindicated when
diverticulitis is present because of the risk of rupturing the diverticulum.

IBD is a common inflammatory functional bowel disorder also known as spastic bowel, functional
colitis and mucous colitis.
1. The client with IBS asks Nurse June what causes the disease. Which of the following responses
by Nurse June would be most appropriate?
a. “This is an inflammation of the bowel caused by eating too much roughage”
b. “IBS is caused by a stressful lifestyle”
c. “The cause of this condition is unknown”
d. “There is thinning of the intestinal mucosa caused by ingestion of gluten”
2. Which of the following alimentary canal is the most common location for Chron’s disease?
a. Descending colon
b. Jejunum
c. Sigmoid Colon
d. Terminal Ileum
3. Which of the following factors is believed to be linked to Chron’s disease?
a. Diet
b. Constipation
c. Heredity
d. Lack of exercise
...................................................................................................................
...................................................................................................................
...................................................................................................................
1. Answer: C. “The cause of this condition is unknown” 
Rationale: There is no known cause of IBS, and diagnosis is made by excluding all the other
diseases that cause the symptoms. There is no inflammation if the bowel. Some factors exacerbate
the symptoms including anxiety, fear, stress, depression, some foods and drugs but there do not
cause the disease.
2. Answer: d. Terminal Ileum
Rationale: Chronic inflammatory of GI mucosa occurs anywhere from the mouth to anus but most
often in terminal ileum. Inflammatory lesions are local and involve all layers of the intestinal wall.
3. Answer: C. Heredity 
Rationale: The cause is unknown but is thought to be multifactorial. Heredity, infectious agents,
altered immunity or autoimmune and environmental are factors to be considered. Test taking skill:
which does not belong? Options a, b, and d are all modifiable factors.

Diabetic Mellitus is a disorder in which blood sugar level are abnormally high because the body does
not produce enough insulin to meet its needs.Discharge plan of diabetic client include insulin
injection site rotation.You should emphasize that the space between sites should be: A. 2.5 cm B.
4.5 cm C.2.5 in D.4.5 in
Ans: A. 2.5 cm. The 4 main areas for injection are the abdomen,upper arms (posterior surface),thighs(anterior
surface),andg hips but the best site is the abdomen.Systematic rotation of injection site within an anatomic area is
reconended to prevent localized changes in fatty tissue (lipodystrophy).In addition ,to promote consistency in insulin
absorption,the client shld be encouraged to use all available injection shld be administered 0.5 to 1 inch away frm the
previous injection.It is also advised that same area shld be used at the same time of the day.

Skin grafting is a technique in which a section of skin is detached frm its own blood supply and
transferred as free tissue to a recipient site.You are taking care of a burn client who undergoes a
skin grafting procedure.After the procedure,the nurse would most likely expect post-operative pain to
be located at the.
A. Recipient site 
B.Heterograft
C.Donor site 
D. Xenograft site
Ans. C Donor site.After the client underwent skin grafting,the pain is felt where the skin was taken frm and that is in
the donor site. The recipient site is the one that has been damaged,therefore receives the new skin.Both donor and
recipient patients may manifest signs of infèction,and as a result,antibiotics may be prescribed.The recipient client
may manifest rejection so steroids may also be prescribed.Xenograft or heterografts skin taken frm animals.

Cholecystitis refers to the acute inflammation of the gall bladder. A client with cholecystitis
experience tenderness and rigidity of the upper right abdomen. Which of the following clients has the
greatest risk for developing cholecystitis?
A. a 28 year old female who has sedentary lifestyle 
B. a 50year old male who has a history of chronic alcoholism 
C. a 80 year old male who has a history of cardiomyopathy 
D. a 45 year old female who has three children and is now on hormone replacement therapy
.
.
.
ANSWER: D. a 45 year old female who has three children and is is now on hormone replacement
therapy.
The risk factors of cholecystitis are woman who is multiparous, taking hormone replacement therapy,
history of gall stones, diabetes mellitus, bacterial infection, prolonged surgery and anesthesia use
and severe trauma.
Category: Immunologic Problem MS
• An HIV-positive patient who has been started on antiretroviral therapy (ART) is seen in the clinic for
follow-up. Which test will be most helpful in determining the response to therapy?
A. Lymphocyte count
B. ELISA testing
C. Western blot analysis
D. Viral load testing
-
Correct answer is letter D 
why?
Rationale: Viral load testing measures the amount of HIV genetic material in the blood, so a
decrease in viral load indicates that the ART is effective. The lymphocyte count which is letter A is
used to assess the impact of HIV on immune function but will not directly measure the effectiveness
of antiretroviral therapy. The ELISA and Western blot tests monitor for the presence of antibodies to
HIB, so these will be positive after the patient is infected with HIV even if drug therapy is effective. 

Mark was recovered from a site of gas tank explosion and was rushed to the nearest hospital. The
paramedics endorsed to the nurse that the patient has full-thickness burns. The nurse anticipates
that the patient she will attend to will be:
A. Anxious to see oneself full of blisters 
B. Extremely sensitive to touch and temperature changes 
C. Uncomfortable due to the hard, leathery feel of the skin 
D. Screaming due to tremendous pain
.
.
.
ANSWER: C
The depth of involvement of a full thickness burn would be all the elements of the skin including the
subcutaneous layer. It is observed that the color ranges from brown to black or marble white. Skin
surface would be charred, dry and there will be leathery tissue formation. The client will not feel any
pain because of the depth of injury as a result grafting is required for the wound to heal.

1. A patient with acute glomerulonephritis with renal insufficiency and generalized edema should
receive what type of diet?
A. High carbohydrates
B. Low carbohydrates
C. High protein
D. High sodium
2. A patient is currently undergoing his initial peritoneal dialysis. NurseAnjanette was able to observe
for an occasional bloody effluent. What will be her priority intervention?
A. Obtain a sample of the effluent and submit it to the laboratory
B. Notify the physician immediately
C. Document her observation
D. Abruptly stop the infusion of the dialysate
...................................................................................................................
...................................................................................................................
...................................................................................................................
ANSWERS:
1. A. High carbohydrates. In a hospital setting, carbohydrates are given liberally to provide energy
and reduce the catabolism of protein. Dietary protein is restricted when renal insufficiency and
nitrogen retention develop. Sodium is also restricted when the patient has hypertension, edema, and
heart failure.
2. C. Document her observation. Bleeding is common during the first few exchanges after a new
catheter insertion because some blood enters the abdominal cavity following insertion. The nurse
should not panic because it is an expected occurrence following the procedure.

The nurse is caring for a 4 year-old admitted after receiving burns to more than 50% of his body.
Which laboratory data should be reviewed by the nurse as a priority in the first 24 hours?
A) Blood urea nitrogen
B) Hematocrit
C) Blood glucose
D) White blood count
The correct answer is A: Blood urea nitrogen Glomerular filtration is decreased in the initial response to severe burns,
with fluid shift. Kidney function must be monitored closely, or renal failure may follow in a few days.

• What is the term for inflammation of the cornea?


A. Conjunctivitis 
B. Corneatitis
C. Scleritis
D. Keratitis
• The main cause of pain discharges in Bacterial Keratitis?
A. Ulcer
B. Infection
C. Virus
D. Trauma
• The FIRST etiology (etiologic agent) of keratitis?
A. Streptococcus pneumoniae 
B. Viridans streptococci 
C. Serratia marcescens
D. Klebsiella pneumoniae
-----
Correct Answers: 
•D
•B
•A
Rationale: KERATITIS is an inflammation of the cornea that sometimes occurs with INFECTION
after viruses, bacteria, or fungi enter the cornea. These microorganisms can enter the eye after
superficial or deep injuries, causing BACTERIAL INFECTION (i.e bacterial keratitis) without having a
loss of tissue (an ulcer) and a cornea may have an ulcer without a bacterial infection which is the
main case of severe pain, discharges and even visual loss. A variety of bacterial species have been
shown to cause keratitis, however the majority of bacterial keratitis incidences are cause by
Staphylococcus aureus, Streptococcus pneumoniae (pneumococcus) and Pseudomonas
aeroginosa. Enterobacteria (Serratia marcescens) comprise the FOURTH most common bacteria
that cause keratitis. While the most frequent streptococcal species involved in bacterial keratitis is
Streptococcus pneumoniae or pneumococcus. If S. pneumonia is the organism infecting the lacrimal
sac, it can spread to the cornea and cause keratitis, esp the cornea has been traumatized.
Pneumococcus keratitis appears to be decreasing as Pseudomonas keratitis is increasing, but
S.PNEUMONIAE REMAINS A SIGNIFICANT ETIOLOGIC AGENT IN KERATITIS. Those 3 etiologic
agents are the most commonly documented gram-positive bacteria.

1. Randy has undergone kidney transplant, what assessment would prompt Nurse Katrina to
suspect organ rejection?
a. Sudden weight loss
b. Polyuria
c. Hypertension
d. Shock
2. The immediate objective of nursing care for an overweight, mildly hypertensive male client with
ureteral colic and hematuria is to decrease:
a. Pain
b. Weight
c. Hematuria
d. Hypertension
3. Matilda, with hyperthyroidism is to receive Lugol’s iodine solution before a subtotal thyroidectomy
is performed. The nurse is aware that this medication is given to:
a. Decrease the total basal metabolic rate.
b. Maintain the function of the parathyroid glands.
c. Block the formation of thyroxine by the thyroid gland.
d. Decrease the size and vascularity of the thyroid gland.
Correct Answer- CAD
1. Answer: (C) Hypertension
Rationale: Hypertension, along with fever, and tenderness over the grafted kidney, reflects acute
rejection.
2. Answer: (A) Pain
Rationale: Sharp, severe pain (renal colic) radiating toward the genitalia and thigh is caused by
uretheral distention and smooth muscle spasm; relief form pain is the priority.
3. Answer: (D) Decrease the size and vascularity of the thyroid gland.
Rationale: Lugol’s solution provides iodine, which aids in decreasing the vascularity of the thyroid
gland, which limits the risk of hemorrhage when surgery is performed.
The physician orders sublingual nitroglycerin for a patient with the diagnosis of angina pectoris
(chest pain related to transient cardiac ischemia). When teaching the patient about this medication,
the nurse should instruct the patient to:
A. "Take only 1 dose of nitroglycerin. If the pain continues, call 911."
B. "Take 1 dose every 3 minutes as often as necessary until pain is relieved."
C. "Double the dose of nitroglycerin 5 minutes after the first dose if there is no relief from pain."
D. "You can repeat the dose of nitroglycerin every 5 minutes for 3 doses. If the pain is unrelieved,
get immediate medical attention."
---
Correct Answer: D
Rationale: Sublingual nitroglycerin has a rapid onset and a relatively short duration of action. 3
doses may be necessary to achieve a desired therapeutic response. However, if pain persists
beyond 15 minutes, it may indicate the presence of an acute cardiac event that requires emergency
medical intervention.
“Your heart is a powerful force. Use it consciously”
Isang basic question lang smile emoticon
Which of the following treatment is a suitable surgical intervention for a client with unstable angina?
A. Percutaneous transluminal coronary angioplasty (PTCA) 
B. Nitroglycerin 
C. Echocardiogram 
D. Cardiac catheterization
.
.
.
ANSWER: A
PTCA can alleviate the blockage and restore blood flow and oxygenation. 
An echocardiogram is a noninvasive diagnosis test. 
Nitroglycerin is an oral sublingual medication. 
Cardiac catheterization is a diagnostic tool – not a treatment.
1. The nurse initiates the client's first hemodialysis treatment. The client develops a headache,
confusion and nausea. These symptoms indicate which of the following potential complications?
A. Disequilibrium syndrome
B. Myocardial infarction
C. Air embolism
D. Peritonitis
2. Which of the following abnormal blood values would not be improved by dialysis treatment?
A. Elevated serum creatinine
B. Hyperkalemia
C. Decreased hemoglobin
D. Hypernatremia
...................................................................................................................
...................................................................................................................
...................................................................................................................
ANSWERS:
1. A. 
Disequilibrium syndrome is common during initial hemodialysis. This results from more rapid
removal of waste products from the blood than from the brain, due to the presence of blood - brain
barrier. Higher concentration of waste products in the brain attracts more fluids. Cerebral edema
occurs, causing increased ICP. To prevent disequilibrium syndrome, initial hemodialysis should be
done for 30 minutes only, then the duration of the procedure will gradually increased.
2. C
Dialysis could not improve hemoglobin levels. The procedure is effective in removing metabolites
like urea and creatinine, edema fluids and excess electrolytes from the blood

1. Propylthiouracil (PTU) is prescribed for a client with Grave's disease to decrease circulating
thyroid hormone. Nurse Lemuel should teach the client to immediately report which of the following
signs and symptoms?
A. Sore throat
B. Painful, excessive menstruation
C. Constipation
D. Increased urine output
2. The nurse asks the client to state her name as soon as he regains consciousness postoperatively
after a subtotal thyroidectomy and at each assessment. The nurse does this primarily to monitor for
signs of which of the following?
A. Internal hemorrhage
B. Decreasing level of consciousness
C. Laryngeal nerve damage
D. Upper airway obstruction
................................................................................................................................................................
................................................................................................................................................................
.........................
ANSWERS:
1. A. Sore throat
Agranulocytosis is a common toxic effect of PTU. This is characterized by fever, skin rash, sore
throat
2. C. Laryngeal nerve damage 
To assess for laryngeal nerve damage after subtotal thyroidectomy, the patient should be asked to
speak every hour. Serves hoarseness of voice or "whispery voice" indicates laryngeal nerve
damage.

When prioritizing care, which of the following clients should the nurseGrace assess first?
A. A 17-year-old clients 24-hours postappendectomy 
B. A 50-year-old client 3 days postmyocardial infarction 
C. A 50-year-old client with diverticulitis 
D. A 33-year-old client with a recent diagnosis of Guillain-Barre syndrome
.
.
.
ANSWER: D
Guillain-Barre syndrome is characterized by ascending paralysis and potential respiratory failure.
The order of client assessment should follow client priorities, with disorder of airways, breathing, and
then circulation. There’s no information to suggest the postmyocardial infarction client has an
arrhythmia or other complication. There’s no evidence to suggest hemorrhage or perforation for the
remaining clients as a priority of care.
Just now

Glaucoma is one of the leading causes of irreversible blindness in the world. Mr. Mark was recently
diagnosed with glaucoma. He went to the hospital to have his eyes checked after having discomfort
around the eyes, blurred vision and loss of peripheral vision. While reviewing the client's medication,
the nurse should question which of the following drug orders?
a. Carbachol 
b. Betaxolol 
c. Atropine sulfate 
d. Epinephrine
.
.
.
ANSWER: C. Atropine sulfate. 
Anticholinergics such as atropine sulfate cause pupil dilation. When pupil dilates it blocks the outflow
of the aqueous humor and increases the intraocular pressure, which can lead to blindness. Betaxolol
is a beta blocker while Epinephrine is an adrenergic agonists. Both Betaxolol and Epinephrine
decreases the aqueous humor production. Carbachol is a cholinergic (miotics) which increases the
outflow of the aqueous humor by constricting the ciliary muscle. None of these are contraindicated in
a client with glaucoma.

3.The client with a history of lung disease is at risk for developing respiratory acidosis.The nurse
asseses this client for which signs and symptoms characteristic of this disorder?
A.Bradycardia and hyperactivity
B.Decreased respiratory rate and depth
C.Headache,restlessness and confusion
D.Bradypnea,dizziness and paresthesias

Which of the following classes of medications protects the ischemic myocardium by blocking
catecholamines and sympathetic nerve stimulation?
a. Beta-adrenergic blockers
b. Calcium channel blockers
c. Narcotics
d. Nitrates

ANSWER: A Beta-adrenergic blockers work by blocking beta receptors in the myocardium, reducing the response to
catecholamines and sympathetic nerve stimulation. They protect the myocardium, helping to reduce the risk of
another infarction by decreasingthe workload of the heart and decreasing myocardial oxygen demand. Calcium
channel blockers reduce the workload of the heart by decreasing the heart rate. Narcotics reduce myocardial oxygen
demand, promote vasodilation, and decreased anxiety. Nitrates reduce myocardial oxygen consumption by
decreasing left ventricular end-diastolic pressure (preload) and systemic vascular resistance (afterload).

he nurse is checking the client's central venous pressure. The nurse should place the zero of the
manometer at the:
A.Phlebostatic axis
B. PMI
C. Erb's point
D.Tail of Spence
Answer A is correct. The phlebostatic axis is located at the fifth intercostals space midaxillary line
and is the correct placement of the manometer. The PMI or point of maximal impulse is located at
the fifth intercostals space midclavicular line, so answer B is incorrect. Erb’s point is the point at
which you can hear the valves close simultaneously, making answer C incorrect. The Tail of Spence
(the upper outer quadrant) is the area where most breast cancers are located and has nothing to do
with placement of a manometer; thus, answer D is incorrect.

 female client tells nurse Karyl that she has been working hard for the last 3 months to control her
type 2 diabetes mellitus with diet and exercise. To determine the effectiveness of the client's efforts,
the nurse should check:
A. Serum fructosamine level. 
B. Fasting blood glucose level. 
C. Urine glucose level. 
D. Glycosylated hemoglobin level.
.
.
.
ANSWER: D
Because some of the glucose in the bloodstream attaches to some of the hemoglobin and stays
attached during the 120-day life span of red blood cells, glycosylated hemoglobin levels provide
information about blood glucose levels during the previous 3 months. Fasting blood glucose and
urine glucose levels only give information about glucose levels at the point in time when they were
obtained. Serum fructosamine levels provide information about blood glucose control over the past 2
to 3 weeks.

A patient is admitted to the clinicak unit after havinh a tracheostomy.When orienting the pt. To the
room. which of the ff explanation would be most important for thw nurse to include?
A.reason for oxygen collar
B.use of call light
C.procedure for suctioning
D.ways to prevent infxn.
Correct answer B. it is important that patient know how to call the nurse.The patient needs to be
aware of how to use the call light. since the patient has tracheostomy he is unble to speak. if ur
answer C. While procedure for suctiining should be explained to the patient it is not impt aspect
WHEN orienting the pt to the room. if ur answer D. prevention infection in a new tracehostomy pt is
primarily the responsiblity of the nurse. it is not most impt when orienting the patient to the room...

A 43-year-old African American male is admitted with sickle cell anemia. The nurse plans to assess
circulation in the lower extremities every 2 hours. Which of the following outcome criteria would the
nurse use?
A.Body temperature of 99°F or less
B. Toes moved in active range of motion
C. Sensation reported when soles of feet are touched
D.Capillary refill of < 3 seconds
Answer D is correct. It is important to assess the extremities for blood vessel occlusion in the client
with sickle cell anemia because a change in capillary refill would indicate a change in circulation.
Body temperature, motion, and sensation would not give information regarding peripheral circulation;
therefore, answers A, B, and C are incorrect.

A 23-year-old client is diagnosed with human immunodeficiency virus (HIV). After recovering from
the initial shock of the diagnosis, the client expresses a desire to learn as much as possible about
HIV and acquired immunodeficiency syndrome (AIDS). When teaching the client about the immune
system, the nurse states that adaptive immunity is provided by which type of white blood cell?
A. Monocyte 
B. Lymphocyte 
C. Neutrophil 
D. Basophil
.
.
.
ANSWER: B
The lymphocyte provides adaptive immunity — recognition of a foreign antigen and formation of
memory cells against the antigen.
Adaptive immunity is mediated by B and T lymphocytes and can be acquired actively or passively.
The neutrophil is crucial to phagocytosis. The basophil plays an important role in the release of
inflammatory mediators. The monocyte functions in phagocytosis and monokine production.

1. As a nurse, what is the primary nursing concern that should be taken note of in taking care of
clients with Aplastic anemia?
A. Vomiting
B. Bleeding tendencies
C. Hyperthermia
D. Pain
.
.
ANSWER: B
Patients with aplastic anemia are vulnerable to problems related to RBC, WBC, and platelet
deficiencies. They should be assessed carefully for signs of infection and bleeding. Specific
interventions are delineated in the sections on neutropenia and thrombocytopenia.
................................................................................................................................................................
......................................................................
2. A patient with pernicious anemia have reposted episodes of balance instability and episodes of
fall due to neurologic involvement. What could be the priority when dealing with these patients?
A. Safety
B. Nutrition
C. Pain
D. Activity intolerance
.
.
ANSWER: A
The nurse needs to pay particular attention to ambulation and should assess the patient's gait and
stability as well as the need for assistive devices (eg, canes, walkers) and for assistance in
managing daily activities. Of particular concern is ensuring safety when position sense, coordination,
and gait are affected. Physical and occupational therapy referrals may be needed.
................................................................................................................................................................
......................................................................
3. A patient has sickle cell anemia. The nurse knowing the background of this disease, is well aware
that the priority for this disorder is
A. Safety
B. Nutrition
C. Pain
D. Activity intolereance
.
.
ANSWER: C
A significant issue is pain management. The incidence of painful sickle cell crises is highly variable;
many patients have pain on a daily basis. The severity of the pain may not be enough to cause the
patient to seek assistance from health care providers but severe enough to interfere with the ability
to work and function within the family. Acute pain episodes tend to be self-limited, lasting hours to
days. If the patient cannot manage the pain at home, intervention is frequently sought in the acute
care setting, usually at an urgent care facility or emergency department

Nurse Reviecca is assessing a male client with heart failure. The breath sounds commonly
auscultated in clients with heart failure are:
A. Friction rubs 
B. Tracheal 
C. Coarse crackles 
D. Fine crackles
.
.
.
ANSWER: D
Fine crackles are caused by fluid in the alveoli and commonly occur in clients with heart failure.
Tracheal breath sounds are auscultated over the trachea. Coarse crackles are caused by secretion
accumulation in the airways. Friction rubs occur with pleural inflammation.

The client admitted with angina is given a prescription for nitroglycerine. The client should be
instructed to:
A.Replenish his supply every 3 months
B. Take one every 15 minutes if pain occurs
C. Leave the medication in the brown bottle
D.Crush the medication and take with water
Correct Answer C. Nitroglycerine should be kept in a brown bottle (or even a special air- and water-
tight, solid or plated silver or gold container) because of its instability and tendency to become less
potent when exposed to air, light, or water.

The adrenal cortex is responsible for producing which substances?


A. Norepinephrine and epinephrine 
B. Glucocorticoids and androgens 
C. Catecholamines and epinephrine 
D. Mineralocorticoids and catecholamines
.
.
.
ANSWER: B
The adrenal glands have two divisions, the cortex and medulla. The cortex produces three types of
hormones: glucocorticoids, mineralocorticoids, and androgens. The medulla produces
catecholamines — epinephrine and norepinephrine.

The nurse is assessing a patient with an abnormal spinal curvature. If curvature exceeds 65
degrees, serious complications may occur. The nurse should assess which region of the spine?
A. Sacral
B. Lumbar
C. Thoracic
D. Cervical
---------------------------------------------
Answer is C
Rationale:
•A curvature of 65 degrees or more in the thoracic spine can cause cardiopulmonary problems due
to impaired chest expansion. If the curvature exceeds 100 degrees, complications can become life-
threatening.
•Life-threatening complications are not generally caused by curvatures of the sacral, lumbar, or
cervical spine

A 37 year old woman diagnosed to have hemothorax. The physician scheduled her for
thoracentesis. Before performing thoracentesis, what must the nurse first ascertain to secure?
A. The patient should be informed about the nature of the procedure.
B. Position the patient comfortably
C. Have a chest radiograph ordered and completed.
D. Inform patient of the importance to remain immoblie
.
.
.
ANSWER C.
Posteroanterior and lateral chest x-ray films are used to located fluid and air in the pleural cavity and
to aid in determining the puncture site.

Which of the following drugs is administered to minimize respiratory secretions preoperatively?


Select one:
A. Valium (diazepam)
B. Atropine sulfate
C. Phenergan (promethazine)
D. Demerol (Meperidine)

Ansswer: B

1. After total laryngectomy surgery,the client has a feeding tube.The primary rationale for tube
feedings is to
A. Meet the fluid and nutritional needs of the client
B. Prevent aspiration
C. Prevent fistula formation 
D. Maintain an open airway
2. Complications associated with a tracheostomy tube include
A. Decrease cardiac output
B. Damage to the laryngeal nerve
C. Pneumothorax
D. Acute respiratory distress syndrome(ARDS)
Correct answers -- AB
Rationale:
1. A. The goal of postoperative care is to maintain physiologic integrity .Therfore,inserting a feeding
tube is a strategy to ensure d fluid n nutritional needs of d client as s surgical site is healing.The
feeding tube does not help prevent aspiration by preventing ingested fluid from leaking through d
wound into s trachea bfore healing occurs;however,d primary rationale is to meet d client's nutritional
n fluid needs.A TEF is a rare complication of total laryngectomy n may occur if radiation therapy has
compromised wound healing.A feeding tube does not help maintain an open airway.
2. B. Tracheostomy tubes carry several potential complications, including laryngeal nerve
damage,bleeding n infection.

1. When the client's common bile duct is obstructed, the nurse should evaluate the client for signs of
which of the following complications?
A. Respiratory distress
B. Circulatory overload
C. Urinary tract infection
D. Prolonged bleeding time
.
.
ANSWER: D
Prolonged bleeding time is a problem if there is biliary obstruction. To absorb Vitamin K in the colon,
there should be presence of colonic bacteria and bile pigment. Decreased or absent bile pigment in
the colon causes inability to absorb Vitamin K. Therefore bleeding may occur.
-------------------------------------------------------------------------------------------------------------------------------------
-------------------------------------------------------------------------------------------------
2. A client undergoes a traditional cholecystectomy and choledochotomy and returns from surgery
with a T-tube. To evaluate the effectiveness of the T-tube, the nurse should understand that the
primary reason for the T-tube is to accomplish which of the following goals?
A. Promote wound drainage
B. Provide a way to irrigate the tract
C. Minimize the passage of bile into the duodenum
D. Prevent the bile into the duodenum.
.
.
ANSWER: C
T-tube is placed in the common bile duct to minimize be passage of bile into the duodenum. It is also
placed to prevent leakage of bile into the peritoneum and to maintain patency of the common bile
duct.
-------------------------------------------------------------------------------------------------------------------------------------
-------------------------------------------------------------------------------------------------
3. A client undergoes a laparoscopic cholecystectomy. Which of the following dietary instructions
would the nurse give the client immediately after surgery?
A. "You cannot eat or drink anything for 24 hours."
B. "You may resume your normal diet the day after surgery."
C. "Drink liquid today and eat lightly for a few days."
D. "You can progress from a liquid to a bland diet as tolerated."
.
.
ANSWER: C
Once client who had undergone laparoscopic cholecystectomy is awake, he may take fluids, then
gradually resume food intake.

A patient is admitted to the hospital and the physician diagnoses him with 4th degree burns. The
nurse would expect the patient's wounds to be:
A. Painful
B. Red
C. Moist
D. Black
E. White
----------------------------------------------
Answer is D
Rationale:
•4th degree: Black, eschar, dry, painless, no blanching.
•Superficial (1st degree): Red, dry, painful.
•Superficial partial thickness (2nd degree): Red, clear blisters, moist, painful, blanches.
•Deep partial thickness (2nd degree): Red and white, bloody blisters, moist, painful, less blanching.
•Full thickness burn (3rd degree): White/Brown, dry, leathery, painless, no blanching.

1. The nurse administers Digoxin to the client with congestive heart failure. The nurse should first
check which of the following?
A. Pulse rate
B. Heart rate
C. Blood pressure
D. Respiratory rate
.
ANSWER. B
The nurse must check the HEART RATE first before digoxin administration. Digoxin slows down
heart rate (negative chronotropic effect). If the heart rate is below 60bpm, the nurse withholds the
medication because it causes bradycardia. If heart rate is 120 bpm and above, the nurse withholds
the medication because it can cause rebound tachycardia
................................................................................................................................................................
......................................................................
2. Which of the following signs and symptoms would most likely be experienced by a client with
right-sided congestive heart failure?
A. Dyspnea, crackles, ankle edema
B. Engorged neck vein, enlarged liver, weight gain
C. Hemoptysis, hypertension, pallor
D. Syncope, hemolytic anemia, clubbing of fingers
.
.
ANSWER: B
Engorged neck vein, enlarged liver, weight gain are signs and symptoms experienced by the client
with right-side congestive heart failure. The signs and symptoms of right-sided congestive heart
failure are due to damming back of blood in the venous circuit.
A, C and D are incorrect. Although ankle edema is experienced by the client, dyspnea and crackles
are manifestations of left sided CHF. Hemolytic anemia occurs in right-sided CHF, but syncope and
clubbing of fingers are manifestations of left-sided CHF.
................................................................................................................................................................
......................................................................
3. Which of the following results of enzyme studies does not indicate the presence of myocardial
infarction (MI)?
A. Elevated CK-MB
B. Elevated SGOT (AST)
C. Elevated LDH
D. Elevated SGPT (ALT)
.
.
ANSWER: D
Elevated SGPT does not indicate the presence of MI. The myocardium does not produce SGPT.
Only liver cells produce SGPT.
A, B and C are enzyme studies that indicate presence of MI

A client is undergoing hemodialysis and becomes hypotensive. The nurse avoids taking which of the
following contraindicated actions?
A. Raising the clients legs and feet
B. Preparing to administer a 250ml normal saline solution
C. Increasing the blood flow from the client into the dialyzer
D. Checking the clients weight and reassessing blood pressure
ANSWER C
to treat hypotension during hemodialysis the nurse raises the clients feet and legs to enhance
cardiac return. A normal saline bolus of up to 500ml may be given to increase circulating volume.
The nurse would check the clients weights and reassess the blood pressure.

The nurse is caring for a client with a newly applied leg cast. The nurse prevents the development of
compartment syndrome by
A. Elevating the limb and applying ice to the affected leg
B. Elevating the limb and covering the limb with bath blankets
C. Keeping the leg horizontal and applying ice to the affected leg
D. Placing the leg in a slightly dependent position and applying ice
ANSWER A
compartment syndrome is prevented by controlling edema. This is achieved most optimally with the
use of elevation and the aplication of ice. The use of bath blankets or a dependent or horizontal leg
position will not prevent this syndrome.
A client is complaining of low back pain that radiates down the left posterior thigh. The nurse further
assesses the client to see if the pain is worsened or aggravated by: 
A.Bed rest
B.Bending or lifting
C.Ibuprofen(Motrin)
D. Application of Heat
BRationale:* Low back pain that radiates into one leg(sciatica) is consistent with herniated lumbar disk. The nurse
assesses the client to see whether the pain is aggravated by events that increase intraspinal pressure, such as
Bending, Lifting, sneezing and coughing or by Lifting the Leg straight up while supine (straight leg-raising test).*Bed
rest, heat(or sometimes ice), and NSAIDs usually relieve back pain, whereas Bending , lifting and straining aggravate
it.

Situation: Tony, a client with leukemia, is in clinic for her routine check-up.
1.which of the ff is unlikely when assessing tony? A. Small abdomen B. Bruises and petechiae C.
Increased WBC count D. Dyspnea during exercise.
2. Tony experiences nasal bleeding. The client should be instructed to? A. Lie supine with neck
extended B. Sit upright, leaning slightly forward C. Blow his nose and then put lateral pressure on his
nose D. Hold his nose while bending forward at the waist

Ans: AB A- small abdomen- leukemia is abnormality of blood cells (rbc- dyspnea is a sign of anemia, wbc- unusually
high especially the young ones, platelets- bruises) B-to prevent aspiration

1. When teaching the client with myocardial infarction (MI), Nurse Loveexplains that the pain
associated with MI is caused by:
A. Left ventricular overload
B. Impending circulatory collapse
C. Extracellular electrolyte imbalances
D. Insufficient oxygen reaching the heart muscle
2. Which of the following is an uncontrollable risk factor that has been linked to the development of
coronary artery disease (CAD)?
A. Exercise
B. Obesity
C. Gender
D. High-cholesterol level
3. A client with angina pectoris ask Nurse Reviecca "What information does ECG provide?" Nurse
Reviecca would appropriately respond that ECG gives information about the...
A. Electrical conduction of the myocardium
B. Oxygenation and perfusion of the heart
C. Contractile status of the ventricles
D. Physical integrity of the muscle
4. How should the nurse instruct the client with angina pectoris to take sublingual nitroglicerine when
chest pain occurs?
A. Take one tablet every 2 to 5 minutes until pain subsides
B. Take one tablet and rest for 10 minutes. Call the physician if the pain persists after 10 minutes.
C. Take one tablet, then an additional tablet every 5 minutes for a total of three tablets. Call the
physician if pain persists after three tablets
D. Take one tablet. If pain persists after 5 minutes, take two tablets. If pain still persists 5 minutes
later, call the physician
5. The client will undergo left-sided cardiac catheterization. Which of the following should not be
included in patient teaching?
A. "You will experience warm sensation as the contrast medium is injected."
B. "You will receive anesthesia"
C. "You will be placed in a tunnel-like device."
D. "You have to tell me if you have allergy to seafoods."
................................................................................................................................................................
................................................................................................................................................................
.........................
Correct Answers: 1)D 2)C 3)A 4)C 5)C
1. D. Insufficient oxygen reaching the heart muscle, causes increased lactic acid production due to
anaerobic metabolism. Lactic acid irritates nerve endings in the myocardium, thereby causing chest
pain.
A, B and C are not causes of chest pain in MI
2. C. Gender has been linked to coronary artery disease and is an uncontrollable or non-modified
risk factor
A, B and D are incorrect. Exercise, obesity and high cholesterol levels are all controllable or
modifiable factors.
3. A. Electrical conduction of the myocardium. ECG directly reflects the transmission of electrical
cardiac impulses throughout the heart.
B, C and D are incorrect. ECG does not give information about oxygenation and perfusion of the
heart. Contractility of the ventricles and physical integrity of the heart muscle may possibly be
evaluated indirectly, but not directly
4. C. Take nitroglycerine at a maximum dose of 3 tablets at 5-minute interval. If the chest pain
persists after three tablets, suspect MI. Then the nurse should call the physician.
A, B and D are incorrect instructions in nitroglycerine therapy
5. C. "You will be placed in a tunnel-like device", should not be included in a patient teaching for
cardiac catheterization. Cardiac catheterization involves radiographic studies of the heart structures
following administration of contrast medium through a catheter.
A, B and D are instructions/information to be included when the client undergoes cardiac
catheterization.

Patient Karyl is suspected of having sickle cell anemia. Having ran through tests, which of the
following results would be considered as one of the aids to the confirmatory factors of her disorder?
A. BUN of 10mg/dl
B. ECG result of normal sinus rhythm
C Hemoglobin level of 7 g/dl
D. Hematocrit level of 37%
ANSWER: C
Symptoms of sickle cell anemia vary and are only somewhat based on the amount of HbS.
Symptoms and complications result from chronic hemolysis and thrombosis. The sickled RBC's have
a shortened life span. Patients are always anemic, usually with hemoglobin values of 7 to 10 g/dl. 
Other options fall under the normal ranges

Nurse Raymond is evaluating the medical records of four clients. Which client does the nurse
evaluate to be at greatest risk for syndrome of inappropriate antidiuretic hormone (SIADH)?
A. Dan, a client with a thyroidectomy 
B. Vincent, a client with oat cell carcinoma of the lung 
C. Ed, a client with polycystic kidney disease 
D. Luis, a client with a fractured hip
.
.
.
ANSWER: B. Vincent, a client with oat cell carcinoma of the lung. 
Conditions associated with syndrome of inappropriate antidiuretic hormone (SIADH) include oat cell
carcinoma of the lung, trauma or tumors of the CNS, stroke, asthma, and COPD and drugs such as
psychotropics, oral hypoglycemic, and chemotherapeutic agents. Polycystic kidney disease is a risk
factor for diabetes insipidus. SIADH is a condition of an oversecretion of ADH. Diabetes insipidus is
an undersecretion of ADH

Polycythemia vera is a malignant disease that progresses in severity over time. Conservative
treatment of repeated phlebotomies ( two to five times per week) can prolong life for 10 to 20 years. 
Roma, a client being phlebotomized for polycythemia vera asks NurseGrace what the is purpose of
phlebotomy. The appropriate reply of Nurse Grace is:
A. To increase viscosity of the blood 
B. To help treat the anemia and prevent it from coming back 
C. To decrease the blood volume 
D. To increase the coagulation factors
.
.
.
ANSWER:C, To decrease the blood volume. 
Polycythemia vera causes an increase in blood cells as a result there is an increase in the blood
volume and blood viscosity. To manage this phlebotomy is performed. Phlebotomy is the blood
drawing with removal of the client's RBCs to decrease the number of RBCs and reduce blood
viscosity.
A phlebotomy would remove blood that can cause anemia, but it does not treat anemia. 
The client with polycythemia vera should also be encouraged to increase fluid intake to decrease the
viscosity of the blood thus preventing clot formation. 
Removing blood does not increase blood thickness
Mr. Cunanan, a client with nasoenteric tube feeding suddenly developed coughing, dyspnea, and
cyanosis. Upon auscultation, there are crackles in the lungs. Nurse Kenneth’s immediate response
are all but one:
A. Prepare possible initiation of antibiotics 
B. Position client on side 
C. Suction nasotracheally and oral tracheally 
D. Consult physician immediately to order chest x-ray examination.
................................................................................................................................................................
................................................................................................................................................................
.........................
ANSWER: A. Prepare possible initiation of antibiotics
It is a part of delayed response wherein there is already fever. All the other options are immediate
responses on aspiration of stomach contents in the respiratory tract

Hypertension is known as the silent killer.this phrase is associated with the fact that hypertension
often goes undetected until symptoms of other systems failures occur. This may occur in the form of
A. CVA
B. Liver disease
C. M.I
D. Pulmonary disease
Ans: A daw

A fracture is a break in the continuity of bone that is caused by any outside force applied to the
bone. 
Nurse Joseph is teaching a class on fracture healing to a group of nursing students. Which of the
following stages of fracture healing should the nurse include in the class?
A. The reparative stage new bone are formed by osteoblasts and dead bone are destroyed by
osteoclasts 
B. Crepitus is a result of extravasation of blood into the subcutaneous tissue 
C. Age and immobilization are not factors to be considered in fracture healing 
D. The postinjury phase is characterized by bleeding and inflammation at the site of injury
.
.
.
ANSWER: A.
Fracture healing progresses over three phases: the inflammatory phase, bleeding and inflammation
at the site of injury initiates this phase. The second phase is the reparative phase, osteoblasts helps
in new bone formation while the osteoclasts destroy the dead bone and helps in the new bone
synthesis. collagen fromation and calcium deposition continues in this stage. the last phase is the
remodeling phase, where excess callus is removed and new bone is laid along the fracture line. then
the fracture site is calcified hence, a new bone is reunited
48. A 72 year-old client with osteomyelitis requires a 6 week course of intravenous antibiotics. In
planning for home care, what is the most important action by the nurse? A) Investigating the client's
insurance coverage for home IV antibiotic therapy B) Determining if there are adequate hand
washing facilities in the home C) Assessing the client's ability to participate in self care and/or the
reliability of a caregiver D) Selecting the appropriate venous access device
-------------------------------------------------------------------------------------------------------------------------------------
-------------------------------------------------------------------------------------------------------------------------------------
-------------------------------------------------------------------------------------------------------------------------------------
----------------------------------------------------------------------------: The correct answer is C: Assessing the
client''s ability to participate in self care and/or the reliability of a caregiver The cognitive ability of the
client as well as the availability and reliability of a caregiver must be assessed to determine if home
care is a feasible option.

46. An 80 year-old client admitted with a diagnosis of possible cerebral vascular accident has had a
blood pressure from 180/110 to 160/100 over the past 2 hours. The nurse has also noted increased
lethargy. Which assessment finding should the nurse report immediately to the health care provider?
A) Slurred speech B) Incontinence C) Muscle weakness D) Rapid pulse
-------------------------------------------------------------------------------------------------------------------------------------
-------------------------------------------------------------------------------------------------------------------------------------
-------------------------------------------------------------------------------------------------------------------------------------
-------------------------------------------------------------------------------------------------------------------->>>>>>>The
CORRECT ANSWER is A: Slurred speech Changes in speech patterns and level of conscious can
be indicators of continued intercranial bleeding or extension of the stroke. Further diagnostic testing
may be indicated.

Serious side effects occur with aggressive chemotherapy including alopecia, mucositis, nausea and
vomiting. Because a client receiving chemotherapy is at risk for developing mouth sores,
nurse Jon instructs the client to:
A. Rinse the mouth with saline solution several times a day 
B. Rinse the mouth with a glycerin based mouthwash several times a day
C. Brush the teeth and gums vigorously to remove plaque 
D. Eat citrus fruits several times a day
.
.
.
ANSWER: A. Rinse the mouth with saline solution several times a day. 
Rinsing the oral mucosa with saline ( or a salt and soda solution) helps to cleanse and protect the
tissue from breakdown. Brushing vigorously and eating citrus products can irritate the oral mucosa.
Over-the-counter mouthwashes containing alcohol can be drying and are usually discouraged.

Nurse Reviecca is evaluating the medical records of four clients. Which client does the nurse
evaluate to be at greatest risk for syndrome of inappropriate antidiuretic hormone (SIADH)?
A. Grace, a client with a thyroidectomy 
B. Jane, a client with oat cell carcinoma of the lung 
C. Love, a client with polycystic kidney disease 
D. Luis, a client with a fractured hip
.
.
.
ANSWER: B. Jane, a client with oat cell carcinoma of the lung. Conditions associated with syndrome
of inappropriate antidiuretic hormone (SIADH) include oat cell carcinoma of the lung, trauma or
tumors of the CNS, stroke, asthma, and COPD and drugs such as psychotropics, oral hypoglycemic,
and chemotherapeutic agents. Polycystic kidney disease is a risk factor for diabetes insipidus.
SIADH is a condition of an oversecretion of ADH. Diabetes insipidus is an undersecretion of ADH

A client who went abdominal surgery who has a nasogastric tube (NGT) in place begins to complain
of abdominal pain that he describe as "feeling full and uncomfortable". Which assessment should
nurse Jane perform first?
A. Measure abdominal girth
B. Auscultate bowel sounds
C. Assess patency of the NGT
D. Assess vital signs
.
.
.
ANSWER: C
When an NGT is no longer patent, stomach contents in the stomach giving the client a sensation of
fullness. The nurse should begin by assessing patency of the NGT. The nurse can measure
abdominal girth, auscultate bowels, and assess vital signs, but she should check NGT patency first
to help relieve the client's discomfort.

Nurse Kathleen know that the late sign of compartment syndrome is?.


A.paresthesia
B.tissue around the distal area is pale, dusky and edematous
C.pain with passive movement
D.pulselessness
Ans: D

A nurse is educating a postpartum client about various contraceptive methods. Which of the
following client statements indicates correct understanding?
1. “Water-soluble lubricants should be used with latex condoms”
2. “A diaphragm must be refitted every year and with weight gain of five pounds”
3. “The vaginal ring (Nuvaring) has to be fitted and replaced every three months”
4. “The basal body temperature (BBT) method can be influenced by stress and alcohol”
5. “Progestins (Depo-Provera) is given as an injection every two years and cannot be taken when
breastfeeding”
A. 1, 3 and 4
B. 1 and 4 only
C. 2 and 4 only
D. All except 5
----------------------------------------------
CORRECT ANSWER is B
RATIONALES:
A. CORRECT: This is a correct statement. Only water-soluble or silicone lubricants should be used
with latex condoms to avoid condom breakage. Oil-based lubricants weaken the integrity of the
condom.
B. A diaphragm does not need to be refitted every year or with a weight change of five pounds. It
must be refitted by the provider and replaced every 2 years, when there is a 7kg (15 lb) weight
change, or 20% weight fluctuation, and after each pregnancy. It should be left in place at least 6
hours after intercourse.
C. The vaginal ring does not have to be fitted. The ring is worn for 3 weeks, with one week without a
ring. Insertion of the new ring should be on the same day of the week each month.
D. CORRECT: This is a correct statement. The basal body temperature (BBT) method can be
influenced by many variables including stress, alcohol, fatigue, illness, and sleep patterns. The
temperature is taken before getting out of bed each morning.
E. Progestins (Depo-Provera) is given as an intramuscular injection every 11 to 13 weeks, not every
two years. Breastfeeding is not contraindicated. The injection is given within five days of delivery
unless breastfeeding. It breastfeeding exclusively the injection is given at the sixth week postpartum
when the milk supply is well established.

The nurse is performing the initial assessment of a client with asthma at the beginning of the shift.
The client has oxygen running at 2 liters per minute per nasal cannula. Which assessment finding
would the nurse be most concerned about?
A. Crackles at the base of the lungs on auscultation
B. Pulse oximetry reading of 89%
C. Excessive thirst with a dry cracked tongue
D. Rapid shallow respirations with intermittent wheezes
The answer is D
Rationale:
* The most common symptoms of asthma includeS
tightness in the chest, labored breathing, coughing and wheezing.

Rapid and shallow respirations associated with labored breathing indicate the client is losing the strength required to
breathe.

The intermittent wheezes indicate increased narrowing of the small airways and a worsening condition.

This client requires prompt and aggressive respiratory intervention


to avoid respiratory failure, including bronchodilators
(such as nebulized albuterol), increased oxygen supplementation
to maintain a SpO2 of at least 92%, and anti-inflammatory medications
(such as IV corticosteroids).

A client who has diabetes mellitus and hypertension has a new prescription for propranolol (Inderal).
Which of the following statements should the nurse include during client teaching? “This medication
A. will likely increase your glucose levels.”
B. can mask the early signs of hypoglycemia.”
C. is more effective when taken on an empty stomach.”
D. has been known to cause a non-productive dry, hacky cough.”
Ans: B

A client has bucks extension traction applied to the right leg. The nurse plans which of the following
interventions to prevent complications from the device?

A. Provide pin care once a shift

B. Massage the skin of the right leg with lotion every 8hours

C. Inspect the skin on the right leg at least once every 8hours

D. Release the weights on the right leg for range of motion exercise daily

ANSWER C

bucks extension traction id a type of skin traction. The nurse inspect the skin of the limb in traction at
least once every 8hours for irritation or inflammation. Massaging the skin with lotion is not indicated.
The nurse never releases the weights of traction unless specifically oredered by physician. There are no
pins to care for with skin traction.

Nurses are tasked with the promotion of healthy outcomes for people in times of crisis. This is achieved
through crisis intervention, a brief and focused strategy that aids people to cope adaptively with
stressful events.

The nurse should know that the main goal of nursing care for a client in crisis would be to:

A. develop the client's insight to the problem

B. assist client to utilize adaptive coping mechanisms immediately

C. restore client's physiologic equilibrium

D. schedule follow-up counselling for the client

.
.

ANSWER: C

Restore client's physiologic equilibrium. Crisis intervention is a short-term therapy which aims to return
clients to their precrisis state.

Developing the client's insight to the problem and assisting client to utilize adaptive coping mechanisms
immediately are long-term outcomes which may not be readily achievable during crisis intervention.

Scheduling for follow-up counselling is not a goal of crisis intervention.

A patient w/ pernicious anemia undergo SCHILLING's Test, which type of specimen is collected?
A. Blood
B. Glucose
C. Urine
D. Sweat
ANSWER: C

A man who fell from a 20feet building is determined to have a spinal cord fracture with spinal cord
transaction at the level of C-6. This injury results in which of the following findings?
A. Quadriplegia with loss of respiratory function.
B. Quadriplegia with diaphgramatic breathing and gross arm movements.
C. Bowel and bladder dysfunction
D. Paraplegia with variable loss of intercostals and abdominal muscle.
ANSWER IS "B" a patient with an injury at the level of c6 has quadriplegia with diaphragmatic breathing and gross
arm movements. The patient may also suffer from hypotension and an atinic bladder. 

C3 - C4 compromised respiratory function,

Mark was brought to the emergency department due to a sprain. Based on nurse Grace's knowledge
on anatomy and physiology, which is primarily affected in a sprain?
A. Ligament
B. Tendon
C. Rotator cuff
D. Both A and B
.
ANSWER: A. Ligament
Sprains are excessive stretching of a ligament, usually caused by a twitching motion, such as in a
fall. 
Other options are incorrect.
A client has a recently placed peritoneal dialysis catheter in place. The charge nurse is teaching a
novice nurse how to perform the client’s first dialysate exchange. Which of the following information
should the charge nurse include in the teaching?
A. Anticipate abdominal pain and cramping.
B. Warm the dialysate fluid by placing it in warm water.
C. Expect the dialysate outflow fluid to be slightly bloody.
D. Wear a mask when connecting and disconnecting the tubing.
E. Ensure the dialysate fluid is at room temperature prior to the infusion.
A. All except E
B. A, B and C
C. C only
D. C and D

CORRECT ANSWER is D
_________________________________________________________________________________

RATIONALE:

A. The client should not experience abdominal pain and cramping. If this occurs, the dialysate solution may be too
cold or it can be an indication of a complication such as peritonitis.

B. Warming should be performed using dry heat (e.g., heating pad, incubator, heating cabinet) as opposed to placing
the solution in warm water or a microwave. Placing the solution in warm water increases the chance of bacterial
contamination.

C. CORRECT: The dialysate fluid return can be bloody during the first few exchanges. After that, It should be
colorless or straw-colored and not cloudy.

D. CORRECT: The nurse should wear a mask to avoid contamination with airborne bacteria when connecting and
disconnecting the catheter from the tubing.

E. The dialysate fluid should be warmed to body temperature, not room temperature prior to the infusion. In addition
to preventing pain and cramping, warming the fluid dilates blood vessels which increase the clearance of urea.

A client develops bilateral wheezes, crackles from bases to apices, orthopnea and tachypnea and
the nurse notes the presence of +2pitting edema. The nurse suspects pulmonary edema and notifies
the physician. While awaiting the physicians arrival. The nurse avoids which actions?
A. Elevating the legs
B. Preparing to administer IV morphine sulfate
C. Preparing to administer iv furosemide 
D. Placing the pt. In the high fowlers position
ANSWER A
Elevating the clients legs would rapidly increase venous return to the right side of the heart and
worsen the clients condition. The feet should be in the horizontal position or the client could dangle
at the bedside if the clients condition permits.
Test taking --> use the process of elimination and note the strategic word AVOIDS.

The nurse is preparing a client for cardioversion using anterolateral paddle placement. The nurse
places the conductive gel pads at which areas on the clients chest in preparation for this procedure?
A. Left fourth intercostal space and left fifth intercostal space at midaxillary line
B. Left second intercostal space and left fifth intercostal space at mudaxillary line
C. Right fourth intercostal space and left fifth intercostal space at anterior axillary line
D. Right second intercostal space and left fifth intercostal at anterior axillary line
ANSWER D
anterolateral paddle placement for external counter shock involves placing one paddle at the right
second intercostal space and the other at the fifth intercostal space at the anterior axillary line.

The nurse is caring for a client with a thoracic spinal cord injury. As part of the nursing care plan the
nurse monitors for spinal shock. In the event that spinal shock occurs the nurse anticipates that the
most likely IV fluid to be prescribed would be
A. Dextran
B. 0.9%normal saline
C. 5%dextrose in water
D. 5%dextrose in 0.9%normal saline
ANSWER B
normal saline is an isotonic solution that primarily remains in the intravascular space, increasing
intravascular volume. This iv fluid would increase the clients blood pressure.

A client with large goiter is scheduled for a subtotal thyroidectomy to treat thyrotoxicosis. Which of
the following is the drug given before the procedure in conjunction with an anti- thyroid agent?
A. Levothyroxine
B. Calcium gluconate
C. Lugol's solution
D. All of the above
Correct answer: D
Before thyroidectomy, make sure the pt has followed his preoperative drug regimen which will render the gland to
prevent thyrotoxicosis during surgery. He probably will have received PTU or methimazole, iodine, ca gluconate and
inderal.

1. The physician prescribes a loop diuretics for a client may develop which electrolyte imbalancce?
A.Hypernatremia. B. Hyperkalemia. C. Hypokalemia. D. Hypervolemia. 2. She finds out that some
managers have benevolent-authoritative style of management. Which of the following behaviors will
she exhibit most likely? A. Have condescending trust and confidence in their subordinates. B. Gives
economic and ego awards. C. Communicates downwards to staffs. D. Allows decision making
among subordinates. 3. A female client with a fecal impaction frequently exhibits which clinical
manifestations? A. Increased appetite. B. Loss of urge to defecate. C. Hard, brown, formed stools.
D. Liquid or semi-liquid stools. 4. A male client is admitted and diagnosed with acute pancreatitis
after a holiday celebration of excessive food and alcohol. Which assessment finding reflects this
diagnosis? A. Blood pressure above normal range. B. Presence of crackles in both lung fields. C.
Hyperactive bowel sounds. D. Sudden onset of continuous epigastric and back
pain. ....... ....... ....... ....... ....... ....... Answers. 1. Answer B. Hyperkalemia. A loop diuretic removes
water and, along with it, sodium and potassium. This may result in hypokalemia,hypovolimia,and
hyponatrimia. 2. Answer A. Have condescending trust and confidence in their subordinates.
Benevolent- authoritative managers pretentiously show their trust and confidence to their followers.
3. Answer D. Liquid or semi- liquid stools. Passage of liquid or semi- liquid stools results from
seepage of unformed bowel contents around the impacted stool in the rectum. Clients with fecal
impaction don't pass hard, brown, formed stools because the feces can't move past the impaction.
These clients typically report the urge to defecate( although they can't pass stool) and a decreased
appetite. 4. Answer D. Sudden onset of continuous epigastric pain and back pain. The autodigestion
of tissue by the pancreatic enzymes results in pain from inflammation, edema, and posssible
hemorrhage. Continuous, unrelieved epigastric or back pain reflects the inflammatory process in the
pancreas.

Nurse Mark Joseph is teaching a client with history of atherosclerosis. To decrease the risk of
atherosclerosis, the nurse encourage the client to: A. Avoid focusing on his weight. B. Increase his
activity level. C. Follow a regular diet. D. Continue leading a high- stress lifestyle. 2. Nurse Jhet is
working on a surgical floor. Nurse Jhet must logroll a client following a: A. Laminectomy. B.
Thoracotomy. C. Hemorrhoidectomy. D. Cystictomy. 3. A male client undergone a colon resection.
While turning him, wound dehiscence with evisceration occurs. Nurse Tin first response is to: A. Call
the physician. B. Place a saline-soaked sterile dressing on the wound. C. Take a blood pressure and
pulse. D. Pull the dehiscence closed. 4. Nurse Tim is assessing a male client with heart failure. The
breath sounds commonly auscultated in clients with heart failure are: A. Tracheal. B. Fine crackles
C. Coarse crackles. D. Friction rubs. 5. Nurse Jam is taking a health history of an 84 year old client.
Which information will be most useful to the nurse for planning care? A. General health history for
the past 10 years. B. Current health promotion activities. C. Family history of diseases. D. Marital
status. .............. ....... ....... ....... Answers: 1. Answer B. Increase his activity level. The client should
be incouraged to increase his activity level. Maintaining an ideal weight; following a low cholesterol,
low sodium diet, and avoiding stress are all important factors in decreasing the risk of
atherosclerosis. 2. Answer A. Laminectomy. The client who has had spinal surgery, such as
laminectomy, must be log rolled to keep the spinal column straight when turning. Thoracotomy and
cystectomy may turn themselves or may be assisted into a comfortable position. Under normal
circumstances, hemorrhoidectomy is an outpatient procedure, and the client may resume normal
activities immediately after surgery. 3. Answer B. Place a saline - soaked sterile dressing on the
wound. To prevent tissue drying and possible infection. Then the nurse should call the physician and
take the client's vital signs. The dehiscence needs to be surgically closed, so the nurse should never
try to close it. 4. Answer B. Fine crackles. Fine crackles are caused by fluid in the alveoli and
commonly occur in clients with heart failure. Tracheal breath sounds are auscultated over the
trachea. Coarse crackles are caused by secretion accumulation in the airways. Friction rubs occur
with pleural inflammation. 5. Answer B. Current health promotion activities. Recognizing an
individual's positive health measures is very useful. General health in the previous 10yrs is
important, however, the current activities of an 84yr old client are most significant in planning care.
Family history of disease for a client in later yrs is of minor significance. Marital status information
may be important for discharge planning but is not as significant for addressing the immediate
medical problem.

A client with AIDS will be recieving aerosolized pentamidine isethonate (nebupent) prophylactically
once every 4weeks. The home health nurse visits and instruct the client about the medication. Which
statement by the client indicates a need for further teaching?
A. There are no known side effects of this therapy
B. I may experience some nausea with the inhalation therapy
C. If i have any visual disturbances i need to let the doctor know
D. If i develop a cough or shortness of breath after recieving the inhalation therapy. I need to let a
doctor or nurse know
ANSWER A
side effects associated with this aerodolized therapy include nausea visual disturbances and
shortness of breath. The client needs to inform the health care provider if these side effects occur.
Test taking --> note the strategic word INDICATES A NEED FOR FURTHER TEACHING. These
words indicate a negative event query and the need to select an option that is an incorrect client
statement.

1. A Client with HF is receiving lasix, and lanoxin, when u enter the room to administer the morning
doses, he complaints of anorexia, nausea, and yellow vision, you should do which of the ff. 1st?
A. Adminiater meds
B. Give the lanoxin only
C. Check the morning serum K+ level
D. Check the morning digoxin level
3. A client has an order to be given beclomethasone by the intra nasal route. He also has an order
for nasal decongestant. You plan to?
A. Administer the beclomethasone 15mins. Before.the decongestant.
B. Administer the decongestant 15mins before the beclomethasone
C. Administer the beclometasone immediately before the decongestant
D. Administer the decongestant immediately before the beclomethasone.
3. A client has received atropine sulfate IV during sx procedure. You monitor him for which effect of
the medication in the immediate operative period?
A. Bradycardia
B. Excessive salivation
C. Diarrhea
D. Urinary retention
4. You asses the 12th cranial in the client who sustained CVA. You ask him to?
A. Extend the arms
B. Extend the head towards your arm
C. Extend the tongue
D focus the eye on the object held by you
5 You are listening to the client's breath sound and heat creaking, grating sound.on inspiration and
expiration ovwr the right posterior lobe. You document that he has
A. Crackles
B. Wheezes
C. Rhonchi
D. Pleural friction rub
ANSWER
1. D anorexia, nausea, yellow vision are manifestation of digoxin toxicity
2. B decongestant first, to decongest the congested nose thus facilitating the administration and
absorption of beclomethasone.
3. D atrophine so4 is anticholenergic, urinary retention is the most common side effect.
4. C. 12th cranial nerve is hypoglosal it inervates the motor function of the tongue.
5. D. A is bubbling, B is wheezing C is snooring...

An allergic reaction is a manifestation of tissue injury resulting from interaction between an antigen
and an antibody. In the client's chart the nurse read that the client has an allergy to iodine and
shellfish. The perioperative nurse anticipates that:
a. The surgery will be postponed.
b. Another prep solution will be used instead of betadine.
c. The client may experience hives intraoperatively and rashes will appear postoperatively 
d. The client wil be given claritin prior to surgery
.
.
.
ANSWER: B Another prep solution will be used instead of betadine.
A client who reports an allergy to shellfish and iodine should not have prep solutions with betadine
applied because a severe allergic reaction may result. A different pre-op solution will be chosen by
the surgical team

A client has been taking procainamide. The nurse knows the priority intervention before
administering the medication is to
A. Check the blood pressure and pulse
B. Administer the medication as ordered
C. Obtain a complete blood cell count and liver function studies
D. Schedule the client for a drug level to be drawn 1hour after the dose
ANSEER A
procainamide is an antidysrhythmic medication. Before the medication is administered the clients
blood pressure and pulses are checked. This medication can cause toxic effects and serum blood
levels would be checked before administering the medication.
Test taking --> use the steps of the nursing process . This will direct you to option A . Because it is
the only assessment action.

A client with acute respiratory distress syndrome has an order to be placed on a continous positive
airway pressure(CPAP) face mask. The nurse implements which of the following for this procedure
to be most effective
A. Obtains baseline arterial blood gases
B. Obtains baseline pulse oximetry levels
C. Applies the mask to the face with a snug fit
D. Encourages the client to remove the mask frequently for coughing and deep breathing exercises
ANSWER C
the face mask must be applied over the nose and mouth with a snug fit which is necessary to
maintain positive pressure in the clients airway. The nurse obtains baseline respiratory assesstment
and arterial blood gases to evaluate the effectiveness of therapy, but these are not done to increase
the effectiveness of the procedure.

A nurse admits a client with myocardial infarction to the coronary care unit. The nurse plans to do
which of the following in delivering care to this client?
A. Begin thrombolytic therapy
B. Place the client on continous cardiac monitoring
C. Infuse intravenous fluid at rate of 150ml per hour
D. Administer oxygen at a rate of 6liters per min. By nasal canula
ANSWER B
Standard interventions upon admittance to the CCU as they relate to this question include continous
cardiac monitoring administering oxygen at a rate of 2to 4liters per minute unless otherwise ordered
and ensuring an adequate Iv line insertion of an intermittent lock.
Test taking --> use the process of elimination .eliminate C D because the values related to the rates
of oxygen and iv fluids are high. From the remaining options note the relationship between the
clients diagnosis and option B.
Mario undergoes a left thoracotomy and a partial pneumonectomy. Chest tubes are inserted, and
one-bottle water-seal drainage is instituted in the operating room. In the
postanesthesia care unit Mario is placed in Fowler's position on either his right
side or on his back to
A. Reduce incisional pain.
B. Facilitate ventilation of the left lung.
C. Equalize pressure in the pleural space.
D. Increase venous return
Answer: (B) Facilitate ventilation of the left lung.
Since only a partial pneumonectomy is done, there is a need to promote expansion of this remaining Left lung by
positioning the client on the opposite unoperated side

Nurse Lea is caring for Mark a client who begins to experience SEIZURE while in bed. Which action
should the nurse implement to prevent aspiration? 
a. Position the client on the side with head flexed forward 
b. Elevate the head 
c. Use tongue depressor between teeth
d. Loosen restrictive clothing
.

ANSWER A
A. Positioning the client on one side with head flexed forward allows the tongue to fall forward and
facilitates drainage secretions therefore prevents aspiration.

The nurse is caring for a patient who recently underwent an abdominal aortic aneurysm repair. The
nurse monitors the patient for which complication?
A. Hemorrhage
B. Pulmonary embolism
C. Infection
D. Acute renal failure
Answer 

D. Acute renal failure

Rationale

•During a AAA repair, a stent graft is placed in the aorta to support the lumen and reduce pressure on the aneurysm
sac. If the stent graft is placed wrong, it can block the renal arteries and cause renal failure.

•Hemorrhage is more common before the AAA repair, when the aneurym ruptures.

•Emboli and infection are also complications of a AAA repair, but they are less common than renal failure

A 60 years old post CVA patient is taking TPA for his disease, the nurse understand that this is an
example of what level of prevention?
A Primary
B Secondary
C Tertiary
D Nota
Answer letter C Rationale: the client already had stroke, TPA stands for TRANSPLASMINOGEN ACTIVATOR which
are thrombolytics , dissolving clots formed in the vessels of the brain.We are just preventing COMPLICATIONS here..

A 75 year old woman comes to the emergency care unit. Upon initial assessment, you noticed her
face, lips, and tongue are so swollen that she can hardly talk. She also exhibits difficulty breathing
with inspiratory wheezing. As the nurse attending to the patient, what is your first action?
A. Administer oxygen support via non-rebreather mask
B. Administer Epinephrine immediately as ordered by physician
C. Administer oxygen support via face mask 
D. Assess patient for signs of hypoxemia.

ANSWER: B
Immediate action for patient exhibiting anaphylaxis is to counter the inflammatory response by giving
sympathomimetics such as Epinephrine to induce bronchodilation and vasoconstriction.

The physical examination is a systematic data-collection method that uses observation to detect
health problems. A 40-year-old male client was rushed to the emergency department after falling
from a 30-foot tree. The nurse in the emergency department prioritizes which of the following
physical assessments for this client?....
a. Assess apical pulse and note for any arrhythmias.... 
b. Assess level of consciousness using Glasgow Coma Scale, the pupils for reaction to light and
accommodation, and vital signs....
c. Assess peripheral perfusion of toes, capillary blanch test, pedal pulses and vital signs.... 
d. Inspect the appearance of the head and skull. Assess vital signs...
------The Answer is --B--------- :))))
b. Assess level of consciousness using Glasgow Coma Scale, the pupils for reaction to light and
accommodation, and vital signs.
Ratio: The client may be having a head injury. If the client suffered a head injury, assessment of the
NEUROLOGICAL functioning should be done FIRST. Specific assessments are made in relation to
client complaints; the nurse’s own observation of problems, the client’s presenting problem, nursing
interventions provided, and medical therapies.

A client with MULTIPLE SCLEROSIS asks Nurse Ameah what diet should she eat to slow down the
disease. The correct response of the nurse is:
Choose one answer.
A. “ Eat a diet high in protein” 
B. “Avoid diet high is saturated fat” 
C. “No special diet is required, you just have to eat a well balanced meal” 
D. “Eat foods low in tyramine”
.
.
.
ANSWER: C
There is NO SPECIAL DIET that has been proven effective in the treatment of multiple sclerosis. A
balanced, wholesome diet is sufficient. A balanced diet will supply the body with the nutrients it
needs to keep healthy and prevent excessive weight gain.

1. Marco who was diagnosed with brain tumor was scheduled for craniotomy. In preventing the
development of cerebral edema after surgery, the nurse should expect the use of:
a. Diuretics
b. Antihypertensive
c. Steroids
d. Anticonvulsants
2. Halfway through the administration of blood, the female client complains of lumbar pain. After
stopping the infusion Nurse Hazel should:
a. Increase the flow of normal saline
b. Assess the pain further
c. Notify the blood bank
d. Obtain vital signs.
3. Nurse Maureen knows that the positive diagnosis for HIV infection is made based on which of the
following:
a. A history of high risk sexual behaviors.
b. Positive ELISA and western blot tests
c. Identification of an associated opportunistic infection
d. Evidence of extreme weight loss and high fever
4. Nurse Maureen is aware that a client who has been diagnosed with chronic renal failure
recognizes an adequate amount of high-biologic-value protein when the food the client selected from
the menu was:
a. Raw carrots
b. Apple juice
c. Whole wheat bread
d. Cottage cheese
5. Kenneth who has diagnosed with uremic syndrome has the potential to develop complications.
Which among the following complications should the nurse anticipates:
a. Flapping hand tremors
b. An elevated hematocrit level
c. Hypotension
d. Hypokalemia
6. A client is admitted to the hospital with benign prostatic hyperplasia, the nurse most relevant
assessment would be:
a. Flank pain radiating in the groin
b. Distention of the lower abdomen
c. Perineal edema
d. Urethral discharge
7. A client has undergone with penile implant. After 24 hrs of surgery, the client’s scrotum was
edematous and painful. The nurse should:
a. Assist the client with sitz bath
b. Apply war soaks in the scrotum
c. Elevate the scrotum using a soft support
d. Prepare for a possible incision and drainage.
8. Nurse hazel receives emergency laboratory results for a client with chest pain and immediately
informs the physician. An increased myoglobin level suggests which of the following?
a. Liver disease
b. Myocardial damage
c. Hypertension
d. Cancer
9. Nurse Maureen would expect the a client with mitral stenosis would demonstrate symptoms
associated with congestion in the:
a. Right atrium
b. Superior vena cava
c. Aorta
d. Pulmonary
10. A client has been diagnosed with hypertension. The nurse priority nursing diagnosis would be:
a. Ineffective health maintenance
b. Impaired skin integrity
c. Deficient fluid volume
d. Pain
-CORRECT ANSWERS:C A B D A B C B D A.
1. C-Glucocorticoid (steroids) are used for their anti-inflammatory action, which decreases the
development of edema.
2. A. The blood must be stopped at once, and then normal saline should be infused to keep the line
patent and maintain blood volume.
3. B. These tests confirm the presence of HIV antibodies that occur in response to the presence of
the human immunodeficiency virus (HIV).
4. D. One cup of cottage cheese contains approximately 225 calories, 27 g of protein, 9 g of fat, 30
mg cholesterol, and 6 g of carbohydrate. Proteins of high biologic value (HBV) contain optimal levels
of amino acids essential for life.
5. A. Elevation of uremic waste products causes irritation of the nerves, resulting in flapping hand
tremors.
6. B. This indicates that the bladder is distended with urine, therefore palpable.
7. C. Elevation increases lymphatic drainage, reducing edema and pain.
8. B. Detection of myoglobin is a diagnostic tool to determine whether myocardial damage has
occurred.
9. D. When mitral stenosis is present, the left atrium has difficulty emptying its contents into the left
ventricle because there is no valve to prevent back ward flow into the pulmonary vein, the pulmonary
circulation is under pressure.
10. A. Managing hypertension is the priority for the client with hypertension. Clients with
hypertension frequently do not experience pain, deficient volume, or impaired skin integrity. It is the
asymptomatic nature of hypertension that makes it so difficult to treat.

Nursing Practice 3: Care of Clients with Physiologic and Psychosocial Alterations (Part A)
Neurology Nursing (Neurological Disorders)
A nurse assesses a young adult in the emergency room following a motor vehicle accident. Which of
the following neurological signs is of most concern?
A) Flaccid paralysis
B) Pupils fixed and dilated
C) Diminished spinal reflexes
D) Reduced sensory responses
The correct answer is B: Pupils fixed and dilated 
Pupils that are fixed and dilated indicate overwhelming injury and intrinsic damage to upper brain
stem and is a poor prognostic sign.

Nursing Practice 3: Care of Clients with Physiologic and Psychosocial Alterations (Part A)
Pulmonary Nursing (Respiratory Disorders)
A client has viral pneumonia affecting 2/3 of the right lung. What would be the best position to teach
the client to lie in every other hour during first 12 hours after admission?
A) Side-lying on the left with the head elevated 10 degrees
B) Side-lying on the left with the head elevated 35 degrees
C) Side-lying on the right wil the head elevated 10 degrees
D) Side-lying on the right with the head elevated 35 degrees
The correct answer is A: Side-lying on the left with the head elevated 10 degrees 
Gravity will draw the most blood flow to the dependent portion of the lung. For unilateral chest
disease, it is best to place the healthiest part of the lung in the dependent position to enhance blood
flow to the area where gas exchange will be best. Ventilation would be minimally affected in the right
dependent lung. This position also enhances the drainage of the infected part of the lung. An
elevation of 35 degrees is counterproductive to therapeutic blood flow and the drainage of
secretions.

THROMBOPHLEBITIS
Kenneth, the charge nurse of the NICU notices that the rate of thrombophlebitis has increased within
the past 3 days. He decides to have a meeting with the staff nurses to review with them the methods
of preventing thrombophlebitis. Which of the following actions would not help in decreasing the rate
of thrombophlebitis?
A. change the IV insertion site every 72 hours 
B. anchor the IV catheter or needle securely at the insertion site 
C. use a large vein when irritating fluids or medications are to be given, and dilute sufficiently
irritating drugs before infusion 
D. apply cold compress immediately over the insertion site then followed by warm, moist compress
.
.
.
ANSWER: D
Applying cold compress and then warm, moist compress is a NURSING INTERVENTION when
phlebitis occurs. It does NOT prevent the occurrence of phlebitis. Changing the IV site every 72
hours, use of large veins when irritating fluids or drugs are to be given, proper dilution of irritating
drugs as well as anchoring the IV cannula securely prevents irritation and possible injury of the vein
which can lead to phlebitis.

Mrs. Cherewski is a 78-year-old patient from Poland who has been admitted to the hospital with
advanced colon cancer. She is receiving palliative care at this time. Which of the following questions
would be best for the nurse to ask to obtain information about cultural factors that influence grieving?
a. “How have you coped with other hospitalizations?” 
b. “Tell me about the family that is available to help you.” 
c. “What do you expect will happen to you?” 
d. “What do you believe about death?” Factors to assess regarding cultural and spiritual beliefs
include the following areas/questions to explore: What do you believe about death? Who makes
health care decisions in your family/culture? Tell me about your family’s/culture’s funeral practices:
correct answer D:

A client with spinal cord injury is at risk of developing footdrop. The nurse should use which of the
following as the effective preventive measures.
a. foot board
b. heel protectors
c. posterior splints
d. pneumatic boots
Answer: C
The effective means of preventing footdrop is the use of posterior splints or high-top sneakers. A foot
board prevents plantar flexion but also places the client more at risk for developing pressure ulcers
of the fee. heel protectors protect the skin, but do not prevent footdrop. pneumatic boots prevent
DVT but not footdrop.
Which assessment finding would alert the nurse to the possibility of hemorrhage following intraocular
surgery? A.Eleveted temperature B.Diplopia C.Visual floaters D.Eye pain
,the answer is D,following intraocular surgery,complaints of eye pain are indecative of increased intraocular pressure
secondary to hemorrhage.temperature elevations may occur as a manifestation of an inflammatory response.diplopia
is commom following cataract surgery when one eye has a lens and the other doesn't. visual floaters are symptoms
of retinal detachment..

A nurse is performing an admission assessment on a newborn infant admitted to the nursery with
the diagnosis of subdural hematoma after a difficult vaginal delivery. The nurse should do which of
the following to assess for major symptoms associated with subdural hematoma?
A. monitor for the urine for blood?
B. monitor for the urine output pattern
C. Test for contractures of the extremities
D. Test for equality of extremities when stimulating reflexes.
Answer: D
Rationale: A subdural hematoma can cause pressure on a specific area of the cerebral tissue. This
can cause changes in the stimuli responses in the extremities on the opposite side of the body,
especially if the infant is actively bleeding. Option a and b are incorrect. After delivery an infant
would normally be incontinent of urine. Blood in the urine would indicate abnormal trauma and would
not a result of the hematoma. Option c is incorrect because contracturs would not occur this soon
after delivery.
3. A client is admitted with a tentative diagnosis of congestive heart failure. Which of the following
assessments would the nurse expect to be consistent with this problem? A) Chest pain B) Pallor C)
Inspiratory crackles D) Heart murmur : The correct answer is C: Inspiratory crackles In congestive
heart failure, fluid backs up into the lungs (creating crackles) as a result of inefficient cardiac
pumping.

2008 NP4 Actual NLE Board Question


Situation: Based on studies of nurses working in special units like the intensive care unit and
coronary care unit it is important for nurses to gather as much information to be able to address their
needs for nursing care.
66. Critically ill patient frequently complain about which of the following when hospitalized?
A. Hospital report
B. Lack of blankets
C. Lack of privacy
D. Inadequate nursing staff
67. Who of the following is at greatest risk of developing sensory problem?
A. Female patient
B. Adolescent
C. Transplant patient
D. Unresponsive patient
68. Which of the following factors may inhibit learning in critically ill patients?
A. Gender
B. Medication
C. Educational level
D. Previous knowledge of illness
69. Which of the following statements does not apply to critically ill patients?
A. Majority need extensive rehabilitation
B. All have been hospitalized previously
C. Are physically unstable
D. Most have chronic illness.
70. Families of critically ill patients desire which of the following needs to be met first by the nurse?
A. Provision of comfortable space
B. Emotional support
C. Updated information on the client’s status
D. Spiritual counselling
ANSWERS: 66. C 
67. D 
68. B 
69. B 
70. B

Category: Eye Disorder


May 2014 Board Question (5 items)
Situation: Floyd Mayweather was diagnosed with Acute Close Angle Glaucoma. He is being seen by
Nurse Pacquiao.
1. What specific manifestation would nurse Pacquiao see in Acute close angle glaucoma that he
would not see in an open angle glaucoma?
A. Loss of peripheral vision
B. Irreversible vision loss
C. There is an increase in IOP
D. Pain
2. Nurse Pacquiao knew that Acute close angle glaucoma is caused by:
A. Sudden blockage of the anterior angle by the base of the iris
B. Obstruction in trabecular meshwork 
C. Gradual increase of IOP
D. An abrupt rise in IOP from 8 to 15 mmHg
3. Nurse Pacquiao knows that Aqueous Humor is produce where?
A. In the Optic Disc
B. In the Lateral ventricles
C. In the Choroids
D. In the Ciliary Body
4. The Doctor orders pilocarpine. Nurse Pacquiao knows that the action of this drug is to:
A. Contract the Ciliary muscle
B. Relax the Ciliary muscle
C. Dilate the pupils
D. Decrease production of Aqueous Humor
5. Mr. Floyd Mayweather has undergone eye angiography using an Intravenous dye and
fluoroscopy. What activity is contraindicated immediately after procedure?
A. Reading newsprint
B. Lying down
C. Watching TV
D. Listening to the music
-----
Correct answers: D A D A A
Rationale:
1. There is NO PAIN in open angle glaucoma. A,B,C are all present in both glaucomas including the
low pressure glaucoma. Pain is absent because the increase in intra ocular pressure is not initiated
abruptly. It is gradual and progressive and will lead to unnoticed loss of peripheral vision. Pain is
present in acute close angle glaucoma because there is a sudden closure or narrowing of the canal
of schlemm. Therefore if you will be ask whats/s is common in both, answer IRREVERSIBLE LOSS
OF PERIPHERAL VISION.
2. Sudden blockage of the angle will cause s/s of acute angle closure glaucoma. B and C are all
related to open angle glaucoma. D is insignificant, If the client bends or cough, IOP can increase
from 8 to as much as 30 mmHg but then return again to normal.
3. Aqueous Humor (AH) is produced in the CILIARY BODY. It is filtered by the trabecular meshwork
into the canal of schlemm.
4. When the ciliary muscles contract, pupils constrict and the angle widens causing an increase AH
outflow and decrease IOP. Relaxing the ciliary muscle will cause mydriasis or dilation, it is used as a
pre op meds for cataract surgery and eye examination to better visualize the structures behind the
iris. Other drugs like betaxolol, Azetazolamide and epinephrine are the drugs used to decrease AH
production.
5. The client had an eye angiography. Eye angiography requires the use MYDRIATICS pre-
procedure. It is done by injecting an Intravenous dye and visualizes the flow of the dye through the
fluoroscopy along the vessels of the eye. This is to assess macular degeneration or
neovascularizations (proliferation of new vessels to compensate for continuous rupture and
aneurysms of the existing vessels) Mydratics usually takes 6 hours to a day to wear off. If client uses
a mydratic, his pupil will dilate making it UNABLE to focus on closer objects. Only A necessitates the
constriction of the pupil to focus on a near object, which Mr. Floyd Mayweather's eye cannot perform
at this time.
For more information about Glaucoma, click here: http://goo.gl/SgOfE
"Glaucoma is a thief of sight. Each day it steals from you precious memories. Not of what was, but of
what's to come."

You might also like